Patho Exam 3 Practice Questions

Lakukan tugas rumah & ujian kamu dengan baik sekarang menggunakan Quizwiz!

A mother rushes her toddler into the emergency department stating, "My baby can't breathe." Initial assessment reveals the child is struggling to breathe in an upright position. He has both inspiratory and expiratory stridor and is using his chest muscles to breath. The nurse suspects the child has which of the following acute respiratory infections? A)Croup B)Asthma C)Epiglottitis D)Bronchiolitis

C)Epiglottitis Epiglottitis typically presents with an acute onset of sore throat and fever. The child appears pale, toxic, and lethargic and assumes a distinctive position—sitting up with the mouth open and the chin thrust forward. Symptoms rapidly progress to difficulty swallowing, a muffled voice, drooling, and extreme anxiety. Moderate to severe respiratory distress is evident. There are inspiratory and sometimes expiratory stridor, flaring of the nares, and inspiratory muscle retractions. Croup or acute laryngotracheobronchitis is a viral infection that affects the larynx, trachea, and bronchi. Acute bronchiolitis is a viral infection of the lower airways, most commonly caused by the respiratory syncytial virus. Asthma is a reactive airway disease rather than an infection.

Which of the following lab results strongly suggest an immunologic response in the client with possible rheumatic heart disease?A)Elevated white blood cell count B)Elevated erythrocyte sedimentation rate (ESR) C)Group A (β-hemolytic) streptococcal antibodies D)High C-reactive protein levels

C)Group A (β-hemolytic) streptococcal antibodies The pathology of RF does not involve direct bacterial infection of the heart. Rather, the time frame for development of symptoms relative to the onset of pharyngitis and the presence of antibodies to the GAS organism strongly suggests an immunologic response. It is thought that antibodies directed against the M-protein of certain strains of streptococci cross-react with glycoprotein antigens in the heart, joints, and other tissues to produce an autoimmune response through a phenomenon called molecular mimicry. Elevated erythrocyte sedimentation rate (ESR) is a blood test that can reveal inflammatory activity in your body. The level of CRP rises when there is inflammation throughout the body. Elevated WBC indicates an infection.

Football fans at a college have been shocked to learn of the sudden death of a star player, an event that was attributed in the media to "an enlarged heart." Which of the following disorders was the player's most likely cause of death? A)Takotsubo cardiomyopathy B)Arrhythmogenicright ventricular cardiomyopathy/dysplasia (ARVC/D) C)Hypertrophic cardiomyopathy (HCM) D)Dilated cardiomyopathy (DCM)

C)Hypertrophic cardiomyopathy (HCM) The most frequent symptoms of HCM are dyspneaand chest pain in the absence of coronary artery disease. Syncope (fainting) is also common and is typically postexertional, when diastolic filling diminishes and outflow obstruction increases. Ventricular arrhythmias are also common, and sudden death may occur, often in athletes after extensive exertion. Risk factors for sudden cardiac death among clients with HCM include a family history of syncope or sudden cardiac death, certain mutations, and extreme hypertrophy of the left ventricle. HCM is characterized by a massively hypertrophied left ventricle with a reduced chamber size.

The most recent blood work of a client with adiagnosis of heart failure indicates increased levels of atrial natriuretic peptide (ANP) and brain natriuretic peptide (BNP). What is the most likely effect of these peptides on the client's physiology? A)Water retention B)Increased tubular sodium reabsorption C)Inhibition of the renin-angiotensin-aldosterone system D)Sympathetic nervous stimulation

C)Inhibition of the renin-angiotensin-aldosterone system The NPs inhibit the sympathetic nervous system and the renin-angiotensin-aldosteronesystem, in addition to decreasing tubular sodium and water reabsorption.

Increased upper airway resistance and decreased airflow into the lungs in neonates (0 to 4 weeks of age) can result from: A)Frequent crying B)Sleeping supine C)Nasal congestion D)Productive coughing

C)Nasal congestion The neonate (0 to 4 weeks of age) breathes predominantly through the nose and does not adapt well to mouth breathing. Any obstruction of the nose or nasopharynx may increase upper airway resistance and increase the work of breathing. While sleeping, the neonate's face should remain away from any surface or cloth in order to maintain an open nasal airway.

A client who developed a deep vein thrombosis during a prolonged period of bed rest has deteriorated as the clot has dislodged and resulted in a pulmonary embolism. Which of the following types of shock is this client at risk of experiencing? A)Cardiogenic shock B)Hypovolemic shock C)Obstructive shock D)Distributive shock

C)Obstructive shock Obstructive shock results from mechanical obstruction of the flow of blood through the central circulation, such as the blockage that characterizes a pulmonary embolism.

A child is brought to the emergency department with a respiratory infection. The child is struggling to breath and is very anxious. The health care providers suspect epiglottitis. Which of the following interventions would be a priority? A)Have parents help hold the child down so blood work can be drawn and sent to lab so WBC count can be reviewed. B)Try to get the child to open his mouth so you can put a tongue blade in the back of the throat to look for swelling or pustules. C)Place the child upright in bed and begin preparing for a tracheostomy placement. D)Administer the first dose of an antibiotic vial liquid suspension as soon as possible to begin fighting the infection.

C)Place the child upright in bed and begin preparing for a tracheostomy placement. Epiglottitis is a medical emergency, and immediate establishment of an airway by endotracheal tube or tracheostomy is usually needed. If epiglottitis is suspected, the child should never be forced to lie down because this causes the epiglottis to fall backward and may lead to complete airway obstruction. Examination of the throat with a tongue blade may cause airway spasm and cardiopulmonary arrest and should be done only by medical personnel experienced in intubation of small children. It also is unwise to attempt any procedure, such as drawing blood, which would heighten the child's anxiety because this also could precipitate airway spasm and cause death.

A client has been experiencing increasing fatigue in recent months, a trend that has prompted an echocardiogram. Results of this diagnostic test suggest that the client's end-diastolic volume is insufficient. Which of the following parameters of cardiac performance will directly decrease as a result of this? A)Inotropy B)Cardiac contractility C)Preload D)Afterload

C)Preload Preloadis the volume of blood stretching the heart muscle at the end of diastole and is normally determined mainly by the venous return to the heart. Afterload represents the force that the contracting heart muscle must generate to eject blood from the filled heart. Cardiac contractility, or inotropy, is the contractile performance of the heart.

As a result of hypoxemia and polycythemia, persons with chronic obstructive bronchitis are prone to: A)Breakdown of elastin B)Left-sided heart failure C)Pulmonary hypertension D)Expiratory airway collapse

C)Pulmonary hypertension Hypoxemia causes reflex vasoconstrictionof the pulmonary vessels and further impairment of gas exchange in the lung. Hypoxemia also stimulates red blood cell production, causing polycythemia. As a result, persons with chronic obstructive bronchitis may develop pulmonary hypertension and right-sided heart failure. With breakdown and loss of lung elasticity and hyperinflation of the lungs with emphysema, the airways often collapse during expiration.

A client asks why his blood pressure pills seem to make him go to the bathroom to urinate frequently when they are not water pills (diuretics). Which of the following physiologic processes explains the long-term regulation of blood pressure most accurately? A)Actions of the renin-angiotensin-aldosterone system B)Release of antidiuretic hormone (vasopressin) by the posterior pituitary C)Renal monitoring and adjustment of extracellular fluid volume D)Integration and modulation of the autonomic nervous system (ANS)

C)Renal monitoring and adjustment of extracellular fluid volume Long-term maintenance and control of blood pressure is accomplished primarily through the renal control of fluid balance. These mechanisms function largely by regulating the blood pressure around an equilibrium point, which represents the normal pressure for a given individual. Accordingly, when the body contains too much extracellular fluid, the arterial pressure rises and the rate at which water and sodium are excreted by the kidney is increased. When blood pressure returns to its equilibrium point, water and sodium excretion returns to normal. Hormonal influences, such as those of ADH and the RAA system, and neural controls are utilized in the shorter-term control of blood pressure.

Congenital heart defects can cause a right heart-to-left heart shunting of blood that results in increased: A)Pulmonary blood volume B)Right ventricle workload C)Unoxygenated blood flow D)Right atrial blood volume

C)Unoxygenated blood flow Right-to-left shunts transfer unoxygenatedblood from the right side of the heart to the left side, diluting the oxygen content of blood that is being ejected into the systemic circulation and causing cyanosis. Left-to-right shunts cause recycling of blood through the pulmonary vessels and the right side of the heart, causing increased volume and workload of the right side of the heart and pulmonary circulation.

While lecturing on COPD, the instructor mentions emphysema. The instructor asks the students, "If the client is a smoker, explain the physiology behind cellular changes occurring in the lung which allow destruction of the alveoli." Which students have an accurate response? Select all that apply. A)"Antiprotease production and release is not adequate to neutralize the excess protease production." B)"The capillary beds can no longer bring the cells to the lung to fight off the infection since they are blocked by fatty plaque." C)"There is α1-antitrypsin deficiency, so this enzyme can't protect the lung from damage." D)"The alveolar tissue is being digested by enzymes; therefore, they can't grow back to restore normal ventilation."

A)"Antiprotease production and release is not adequate to neutralize the excess protease production." C)"There is α1-antitrypsin deficiency, so this enzyme can't protect the lung from damage." Emphysema is characterized by a loss of lung elasticity and abnormal enlargement of the air spaces distal to the terminal bronchioles, with destruction of the alveolar walls. One of the recognized causes of emphysema is an inherited deficiency of α1-antitrypsin, an antiprotease enzyme that protects the lung from injury. Cigarette smoke stimulates the movement of inflammatory cells into the lungs, resulting in increased release of elastase and other proteases. In smokers in whom COPD develops, antiprotease production and release may be inadequate to neutralize the excess protease production such that the process of elastic tissue destruction goes unchecked.

A nurse runs into an old high school friend after 20 years.She notes her friend continues to smoke after all these years. The friend asks, "Do you think I sound hoarse?" Upon further assessment, the nurse/friend notes her friend has several warning signs of cancer. Which manifestations would lead to this conclusion? Select all that apply. A)"I seem to have some difficulty swallowing food... this is new for me." B)"I seem to have more trouble holding my urine than I use to." C)"Can you feel how large my lymph nodes are on my neck and armpits (axillae)?" D)Pericardial friction rub heard on auscultation.E)Feels subcutaneous emphysema in upper chest area.

A)"I seem to have some difficulty swallowing food... this is new for me." C)"Can you feel how large my lymph nodes are on my neck and armpits (axillae)?" Tumors that invade the mediastinum may cause hoarseness because of the involvement of the recurrent laryngeal nerve and cause difficulty in swallowing because of compression of the esophagus. An uncommon complication called the superior vena cava syndrome occurs in some persons with mediastinal involvement. Interruption of blood flow in this vessel usually results from compression by the tumor or involved lymph nodes. Urinary incontinence, pericardial friction rub, and subcutaneous emphysema are not related to this cancer but can occur as complications from treatments.

A 51-year-old man has been diagnosed with chronic bronchitis after a long history of recurrent coughing. Which of the man's following statements demonstrates a sound understanding of his new diagnosis? A)"If I had quit smoking earlier than I did, I think I could have avoided getting bronchitis." B)"I'm pretty sure that I first caught bronchitis from the person who has the cubicle next to mine at work." C)"I read on the Internetthat I might have got bronchitis because I was born with an enzyme deficiency." D)"I think that I probably could have prevented this if I had got in the habit of exercising more when I was younger."

A)"If I had quit smoking earlier than I did, I think I could have avoided getting bronchitis." Smoking is frequently implicated in the etiology of chronic bronchitis. Infections do not typically initiate the disease, and exercise is not noted to have preventative value. Enzyme deficiency is associated with emphysema, but not bronchitis.

A young, male child is born with severe respiratory failure. Over the course of months, the parents note his body looks swollen. They ask, "Is our baby's kidneys not working right? Why is he so swollen?" The nurse bases his or her reply on which of the following physiological principles? A)"The right side of his heart (cor pulmonale) is not pumping effectively. Blood is backlogging in his body, which is why he is so swollen." B)"We just need to call the physician and ask him to give you a prescription for more water pills." C)"Once we get his oxygenation level back to normal, then maybe his kidneys will receive enough oxygenated blood to filter better." D)"This happens when he has so many secretions in his lungs. Maybe we should try some expectorant to thin his secretions so he can cough them out."

A)"The right side of his heart (cor pulmonale) is not pumping effectively. Blood is backlogging in his body, which is why he is so swollen." The term cor pulmonalerefers to right heart failure resulting from primary lung disease or pulmonary hypertension. The increased pressures and work result in hypertrophy and eventual failure of the right ventricle. The manifestations of cor pulmonale include signs of right-sided heart failure, which include venous congestion, peripheral edema, shortness of breath, and a productive cough, which becomes worse during periods of heart failure. None of the other statements are applicable to these parents' questions

A 29-year-old woman who considers herself active and health conscious is surprised to have been diagnosed with preeclampsia-eclampsia in her second trimester. What should her care provider teach her about this change in her health status? A)"We don't really understand why some women get high blood pressure when they're pregnant." B)"This is likely a result of your nervous system getting overstimulated by pregnancy." C)"Hypertension is a common result of all the hormonal changes that happen during pregnancy." D)"Even though you're a healthy person, it could be that you have an underlying heart condition."

A)"We don't really understand why some women get high blood pressure when they're pregnant." The cause of pregnancy-induced hypertension is largely unknown.

Which of the following individuals is experiencing an immunologic lung disorder affecting ventilation that has caused the formation of a granuloma on chest x-ray? A)A 30-year-old African American man who has been diagnosed with sarcoidosis B)An infant whose routine screening is suggestive of cystic fibrosis C)An elderly, lifelong smoker who has been admitted to hospital with emphysema exacerbation D)A 16-year-old girl who must limit her physical activity to prevent the onset of acute asthmatic attacks

A)A 30-year-old African American man who has been diagnosed with sarcoidosis Sarcoidosis is an example of restrictive lung disease, whereas cystic fibrosis (CF), emphysema, and asthma are considered obstructive.

Which of the following clients would be at highrisk for developing primary varicose veins? Select all that apply. A)A 47-year-old waitress who works 12-hour shifts three or four times/week B)A morbidly obese (>100 poundsoverweight) male who works behind the counter of a convenience store 10 hours/day, 5 days/week C)A 56-year-old male who has been immobile due to back surgery and has developed a deep vein thrombosis D)A Marathon runner who has completed three marathons in the past 3 monthsE)A new peritoneal dialysis client who has been utilizing a home machine and performing dialysis every evening beginning at 8 PM

A)A 47-year-old waitress who works 12-hour shifts three or four times/week B)A morbidly obese (>100 poundsoverweight) male who works behind the counter of a convenience store 10 hours/day, 5 days/week A, BFeedback:Prolonged standing and increased intra-abdominal pressure are important contributing factors in the development of primary varicose veins. Because there are no valves in the inferior vena cava or common iliac veins, blood in the abdominal veins must be supported by the valves located in the external iliac or femoral veins. Immobility may cause DVTs (a secondary cause of varicose veins). Peritoneal dialysis has no effect on the development of varicose veins.

From the following clients, who are at high risk for developing heart failure as a result of diastolic dysfunction? Select all that apply. A)A 48-year-old client with uncontrolled hypertension B)A marathon runner with history of chronic bradycardia whose pulse rate is 46 C)A 57-year-old client with history of ischemic heart disease D)A 70-year-old with enlarged left ventricle due to myocardial hypertrophy

A)A 48-year-old client with uncontrolled hypertension D)A 70-year-old with enlarged left ventricle due to myocardial hypertrophy Conditions that reduce the heart's ability to adequately fill during diastole, such as myocardial hypertrophy and tachycardia, can lead to heart failure. Hypertension remains the leading cause of diastolic dysfunction. Ischemic heart disease is associated with systolic heart failure, or impaired contractile performance. It is normal for athletes, like marathon runners, to have slow pulses.

The nurse would anticipate that which of the following clients would be considered a good candidate for coronary artery bypass grafting (CABG)? A)A 56-year-old with a history of MI experiencing new-onset chest pain and ST elevation B)A 24-year-old auto accident client diagnosed with pericardial effusion and cardiac tamponade C)A 87-year-old client admitted with uncontrolled dilated cardiomyopathy D)A 78-year-old client admitted with increasing fatigue related to aortic stenosis

A)A 56-year-old with a history of MI experiencing new-onset chest pain and ST elevation Coronary artery bypass grafting (CABG) may be the treatment of choice for people with significant coronary artery disease (CAD) who do not respond to medical treatment and who are not suitable candidates for percutaneous coronary intervention. CABG does not address valve disorders, pericardial effusion, or cardiomyopathies.

When educating the parents of a cyanotic infant diagnosed with of tetralogy of Fallot, the nurse will include which of the following statements related to the physiological abnormalities? The infant has: Select all that apply. A)A hole in the ventricular septal B)A small, narrow pulmonary outflow channel C)A large, thick, right ventricular wall D)A very small, narrow aortaE)The pulmonary artery arises from the left ventricle

A)A hole in the ventricular septal B)A small, narrow pulmonary outflow channel C)A large, thick, right ventricular wall Tetralogy of Fallot consists of four associated defects: a ventricular septal defect; dextroposition of the aorta; obstruction or narrowing of the pulmonary outflow channel; and hypertrophy of the right ventricle. Narrowing, or coarctation, of the aorta is not associated with tetralogy of Fallot. Rising of pulmonary arteries from the left ventricle instead of the right is a sign of transposition of the great vessels.

Which of the following clients is at risk for developing acute respiratory failure? A)A middle-aged male diagnosed with amyotrophic lateral sclerosis (ALS) who has pneumonia with low O2saturation B)An elderly female living in senior housing who has been exposed to a "cold" while her grandchildren visited C)A teenager in a high school that has had an increase in student absences due to an outbreak of strep throat D)A toddler in daycare who has been sharing toys with peers before the staff could sanitize properly

A)A middle-aged male diagnosed with amyotrophic lateral sclerosis (ALS) who has pneumonia with low O2saturation Acute respiratory failure may occur in previously healthy persons as the result of acute disease or trauma involving the respiratory system, or it may develop in the course of a chronic neuromuscular or lung disease. ALS is a neurodegenerative disease with various causes. It is characterized by rapidly progressive weakness due to muscle atrophy and muscle spasticity and difficulty in speaking (dysarthria), swallowing (dysphagia), and breathing (dyspnea). A common cold, strep throat, or sharing toys in daycare will not necessarily place one at risk for respiratory failure

The shortness of breath and cyanosis that occur in clients experiencing acute heart failure syndrome are primarily caused by: Select all that apply. A)Accumulation of fluid in the alveoli and airways B)Lung stiffness C)Worsening renal failure D)Myocardial muscle necrosis E)Impaired gas exchange

A)Accumulation of fluid in the alveoli and airways B)Lung stiffness E)Impaired gas exchange Acute pulmonary edema is the most dramatic symptom of AHFS. It is a life-threatening condition in which capillary fluid moves into the alveoli. The accumulated fluid in the alveoli and airways causes lung stiffness, makes lung expansion more difficult, and impairs the gas exchange function of the lung. With the decreased ability of the lungs to oxygenate the blood, the hemoglobin leaves the pulmonary circulation without being fully oxygenated, resulting in shortness of breath and cyanosis. Worsening renal failure and MI may cause volume overload but are more likely secondary causes of chronic heart failure.

The most common causes of left-sided heart failure include: A)Acute myocardial infarction B)Chronic pulmonary disease C)Impaired renal blood flow D)Tricuspid valve regurgitation

A)Acute myocardial infarction The most common causes of left-sided heart failure are acute myocardial infarction and hypertension. Acute or chronic pulmonary disease can cause right heart failure, referred to as cor pulmonale. The causes of right-sided heart failure include stenosis or regurgitation of the tricuspid or pulmonic valves, right ventricular infarction, and cardiomyopathy. Manifestations (rather than causes) of heart failure reflect the physiologic effects of the impaired pumping ability of the heart, including decreased renal blood flow.

Which of the following is a nonmodifiable risk factor for the development of primary hypertension? A)African American race B)High salt intake C)Male gender D)Obesity

A)African American race Hypertension not only is more prevalent in blacks than whites, but also is more severe, tends to occur earlier, and often is not treated early enough or aggressively enough. Blacks also tend to experience greater cardiovascular and renal damage at any level of pressure. High salt intake and obesity are modifiable risk factors for hypertension. Male gender is not identified as a risk factor for hypertension.

Which of the following individuals should be prioritized for receiving a seasonal influenza vaccination? A)An 81-year-old resident of a long-term care facility B)A 19-year-old man who was admitted to a hospital for an appendectomy C)A neonate who was born in a busy, inner-city hospital in late October D)An 86-year-old client whose flu symptoms have required hospitalization

A)An 81-year-old resident of a long-term care facility An older adult who lives in a nursing facility is a high-priority candidate for influenza vaccination. Infants younger than 6 months and persons who are acutely ill should not receive immunizations. Younger hospital clients may be vaccinated but are usually of lower priority than the elderly.

A toddler seems to have a little "cold" and runny nose. At bedtime, he appears to be OK. A few hours later, parents awaken hearing a "tight" coughing sound. They recognize the child is not breathing well, so they rush to the emergency department. On arrival, the nurses suspect bronchial asthma based on which of the following assessment data? Select all that apply. A)Audible wheezing B)"Crowing sound with inspiration" C)Respiratory rate—44 with prolonged exhalation D)Coughing up blood-tinged sputum E)Sitting upright, leaning forward, and using accessory muscles to breathe

A)Audible wheezing C)Respiratory rate—44 with prolonged exhalation E)Sitting upright, leaning forward, and using accessory muscles to breathe Bronchial asthma represents a reversible form of obstructive airway disease caused by narrowing of airways due to bronchospasms, inflammation, and increased airway secretions. Healthy children develop what may seem to be a cold with rhinorrhea, rapidly followed by irritability, a tight and nonproductive cough, wheezing, tachypnea (respiratory rate—44), dyspnea with prolonged expiration, and use of accessory muscles of respiration. Cyanosis, hyperinflation of the chest, and tachycardia indicate increasing severity of the asthma attack. Croup is characterized by inspiratory stridor or a barking cough. The British use the term croup to describe the cry of the crow or raven, and this is undoubtedly how the term originated. Hemoptysis (blood in the sputum) is usually associated with pulmonary emboli or lung cancers to name a few.

Soon after delivery, the mother grabs the nurses arm and states, "Something's wrong...I can't get my breath." Which of the following assessments lead the nurse to suspect the client has had an amniotic emboli travel to the lungs? Select all that apply. A)BP 90/65; pulse 130, irregular; respiratory rate 35, shallow. B)Intercostal traction noted on inspiration. C)+3 pitting edema in lower extremities. D)Trachea has shifted and is no longer midline. E)Productive cough with blood-streaked sputum.

A)BP 90/65; pulse 130, irregular; respiratory rate 35, shallow. E)Productive cough with blood-streaked sputum. The embolism may consist of amniotic fluid that has entered the maternal circulation during childbirth. Persons with moderate-sized emboli often present with breathlessness accompanied by pleuritic pain, apprehension, slight fever, and cough productive of blood-streaked sputum. Tachycardia often occurs to compensate for decreased oxygenation, and the breathing pattern is rapid and shallow. Intercostal traction is usually associated with atelectasis; pitting edema is a sign of right-sided heart failure; trachea shift is a classic sign of a tension pneumothorax.

A 6-hour-old newborn develops a critical respiratory problem and is rushed to the ICU. The ICU nurses suspect the infant has respiratory distress syndrome (RDS) based on which findings? Select all that apply. A)Bluish discoloration of the skin and mucous membranes (central cyanosis) B)Substernal retractions with each breathe C)Periorbitaledema D)Clubbed fingers E)Expiratory grunting

A)Bluish discoloration of the skin and mucous membranes (central cyanosis) B)Substernal retractions with each breathe E)Expiratory grunting Infants with RDS present with multiple signs of respiratory distress, usually within the first 24 hours of birth. Central cyanosis is a prominent sign. Breathing becomes more difficult, and retractions occur as the infant's soft chest wall is pulled in as the diaphragm descends. Grunting sounds accompany expiration. Periorbital edema is usually associated with kidney disease in infants. Clubbed fingers occur over a long period of time (years) in clients with COPD.

A client's primary care provider has added 20 mg of Lasix (furosemide) to his medication regimen to treat his primary hypertension. How does this agent achieve its therapeutic effect?A)By decreasing vascular volume by increasing sodium and water excretion B)By blocking the release of antidiuretic hormone from the posterior pituitary C)By inhibiting the conversion of angiotensin I to angiotensin II D)By inhibiting the movement of calcium into arterial smooth muscle cellsAns:A

A)By decreasing vascular volume by increasing sodium and water excretion Diuretics lower blood pressure initially by decreasing vascular volume (by suppressing renal reabsorption of sodium and increasing sodium and water excretion) and cardiac output. Angiotensin-converting enzyme (ACE) inhibitors block the conversion of angiotensin I, and calcium channels blockers inhibit the movement of calcium into arterial smooth muscle. Common antihypertensives do not act directly on the pituitary.

A client is receiving home care for the treatment of a wound on the inside of her lower leg that is 3 cm in diameter with a yellow wound bed and clear exudate. Assessment of the client's legs reveals edema and a darkened pigmentation over the ankles and shins of both legs. What is this client's most likely diagnosis? A)Chronic venous insufficiency B)Deep vein thrombosis C)Varicose veins D)Peripheral arterial disease

A)Chronic venous insufficiency In contrast to the ischemiacaused by arterial insufficiency, venous insufficiency leads to tissue congestion, edema, and eventual impairment of tissue nutrition (development of a venous ulcer). The edema is exacerbated by long periods of standing. Necrosis of subcutaneous fat deposits occurs, followed by skin atrophy. Brown pigmentation of the skin caused by hemosiderin deposits resulting from the breakdown of red blood cells is common. DVTs, varicose veins, and PAD do not have this symptomatology.

Which of the following children may be considered high risk for developing hypertension? Select all that apply. A)Diagnosed with coarctation of the aorta as an infant B)Recent scan showing a pheochromocytoma C)Has a history of epilepsy with weekly seizures D)Takes cyclosporine daily since a kidney transplant E)Has a history of frequent sinus infections treated with antibiotics

A)Diagnosed with coarctation of the aorta as an infant B)Recent scan showing a pheochromocytoma D)Takes cyclosporine daily since a kidney transplantE)Has a history of frequent sinus infections treated with antibiotics Approximately 75% to 80% of secondary hypertension in children is caused by kidney abnormalities. Coarctation of the aorta is another cause of hypertension in children and adolescents. Endocrine causes of hypertension, such as pheochromocytoma and adrenal cortical disorders, are rare. The nephrotoxicity of the drug cyclosporine, an immunosuppressant used in transplant therapy, may cause hypertension in children after kidney transplantation. Epilepsy and sinus infections do not cause hypertension.

A client is admitted with dilated cardiomyopathy with left ventricular dysfunction. The nurse should assess for which of the following clinical manifestations? Select all that apply. A)Dyspnea B)Orthopnea C)Extreme fatigue with activity D)Excess abdominal fluidE)Fainting

A)Dyspnea B)Orthopnea C)Extreme fatigue with activity The most common clinical manifestations of DCM are those related to heart failure, such as dyspnea, orthopnea, and reduced exercise capacity. Hypertrophic cardiomyopathy (HCM) is characterized by myocardial thickening and abnormal diastolic filling. They experience fainting/syncope. Restrictive cardiomyopathy, in which there is excessive rigidity of the ventricular wall, increases the work of ventricular emptying and causes cardiac hypertrophy. These clients experience excess abdominal fluid (ascites).

At the cellular level, cardiac muscle cells respond to an increase in ventricular volume to the point of overload by: Select all that apply. A)Elongating the cardiac muscle cells B)Thickening of the individual myocytes C)Replicating the myofibrils D)Decreasing the ventricular wall thickness E)Symmetrically widening and lengthening the hypertrophy

A)Elongating the cardiac muscle cells D)Decreasing the ventricular wall thickness At the cellular level, cardiac muscle cells respond to stimuli from stress placed on the ventricular wall by pressure and volume overload by initiating several different processes that lead to hypertrophy. With ventricular volume overload,the increase in wall stress leads to replication of myofibrils in series, elongation of the cardiac muscle cells, and eccentric hypertrophy. Eccentric hypertrophy leads to a decrease in ventricular wall thickness or thinning of the wall with an increase in diastolic volume and wall tension. Production of a symmetric hypertrophy occurs with a proportionate increase in muscle length and width, as occurs in athletes; concentric hypertrophy with an increase in wall thickness, as occurs in hypertension; and eccentric hypertrophy with a disproportionate increase in muscle length, as occurs in dilated cardiomyopathy. When the primary stimulus for hypertrophy is pressure overload, the increase in wall stress leads to parallel replication of myofibrils, thickening of the individual myocytes, and concentric hypertrophy. Concentric hypertrophy may preserve systolic function for a time, but eventually the work performed by the ventricle exceeds the vascular reserve, predisposing to ischemia

An elderly client is admitted with the diagnosis of severe aortic regurgitation. Which of the following client reports support this diagnosis? Select all that apply. A)Exertional dyspnea B)Orthopnea C)Frequent angina D)Paroxysmal nocturnal dyspnea E)Palpitations

A)Exertional dyspnea B)Orthopnea D)Paroxysmal nocturnal dyspnea E)Palpitations As aortic regurgitation progresses, signs and symptoms of left ventricular failure begin to appear. These include exertional dyspnea, orthopnea, and paroxysmal nocturnal dyspnea. Angina is a rare symptom. Tachycardia, occurring with emotional stress or exertion, may produce palpitations, head pounding, and premature ventricular contractions.

A person newly diagnosed with Kawasaki disease in the acute phase will likely have which of the following clinical manifestations? Select all that apply. A)Fever B)Peeling of the skin of the fingers and toes C)Edematous hands and feet D)Bilateral conjunctivitisE)Irritability and lability of mood

A)Fever C)Edematous hands and feet D)Bilateral conjunctivitisE)Irritability and lability of mood The acute phasebegins with an abrupt onset of fever, followed by bilateral conjunctivitis, usually without exudates; erythema of the oral and pharyngeal mucosa with "strawberry tongue" and dry, fissured lips; redness and swelling of the hands and feet; rash of various forms; and enlarged cervical lymph nodes. The other symptoms

The "cold viruses" are rapidly spread from person to person. The greatest source of spread is: A)Fingers B)Sneezing C)Plastic toys D)Eye mucosa

A)Fingers Cold viruses have been found to survive for more than 5 hours on the skin and hard surfaces, such as plastic countertops. Aerosol spread of colds through coughing and sneezing is much less important than the spread by fingers picking up the virus from contaminated surfaces and carrying it to the nasal membranes and eyes. The fingers are the greatest source of spread, and the nasal mucosa and conjunctival surface of the eyes are the most common portals of entry of the virus.

Chronic stable angina, associated with inadequate blood flow to meet the metabolic demands of the myocardium, is caused by: A)Fixed coronary obstruction B)Increased collateral circulation C)Intermittent vessel vasospasms D)Excessive endothelial relaxing factors

A)Fixed coronary obstruction Chronic stable angina is caused by fixed coronary obstruction that produces an imbalance between coronary blood flow and the metabolic demands of the myocardium. Endothelial relaxing factors relax the smooth muscle in the vessel wall and allow increased blood flow; treatment for chronic stable angina is with a vasodilating agent, such as nitroglycerine, that relaxes the vessels and enhances coronary blood flow. Intermittent vessel vasospasms, in conjunction with coronary artery stenosis, cause the vasospastic type of angina. Increased formation of collateral vessels is a compensatory response that allows adequate blood circulation to tissues distal to an obstruction.

In the ICU setting, clients who develop shock need thorough head-to-toe assessments. Which of the following clinical manifestations would alert the health care provider that the client may be developing ischemia associated with gastrointestinal redistribution of blood flow? A)Gastric bleeding B)Nausea and vomiting C)Irritable bowel syndrome D)Copious high-volume diarrhea

A)Gastric bleeding In shock, there is widespread constriction of blood vessels that supply the gastrointestinal tract, causing a redistribution of blood flow that severely diminishes mucosal perfusion. Bleeding is a common symptom of gastrointestinal ulceration caused by shock, with onset usually within 2 to 10 days after the original insult. Nausea is unrelated to ischemic damage; irritable bowel syndrome is stress related. With ischemia, the bowel ceases to function, causing a lack of peristalsis and no fecal output.

Following a dust storm in Arizona, several clients have presented to the clinic complaining of productive cough, fever, and night sweats. The health care provider suspects a fungal infection related to breathing dust. One particular client is critical. They suspect the infection has progressed outside the lung when they observe the client has: Select all that apply. A)Generalized lymph node enlargement B)Urine output decreased to 40 mL/hour C)Requires guaifenesin to cough up sputum D)An enlarged liver via palpation E)Copious bleeding at the site where thelab technician drew some blood

A)Generalized lymph node enlargement D)An enlarged liver via palpation E)Copious bleeding at the site where thelab technician drew some blood The most common manifestations of fungal infections in the lungs are productive cough, fever, night sweats, and weight loss. Characteristically, disseminated disease presents with a high fever, generalized lymph node enlargement, hepatosplenomegaly, muscle wasting, anemia, leukopenia, and thrombocytopenia. Urine output of 40 mL/hour is normal. Guaifenesin is an expectorant used in many lung diagnoses to liquefy secretions. It is not specific to fungal infection

Which of the following would be considered a major cause of secondary hyperlipoproteinemia since it increases the production of VLDL and conversion to LDL? A)High-calorie diet B)Diabetes mellitus C)Bile-binding resin D)Cholesterol ingestion

A)High-calorie diet Obesity with high-calorie intake increases the production of VLDL, with triglycerideelevation and high conversion of VLDL to LDL. Excessive cholesterol intake reduces formation of LDL receptors. Diabetes is associated with high triglycerides and minimal elevation of LDL. Bile salt-binding resin is one treatment used to lower cholesterol

Electrical burns over a large surface area of a client's body have resulted in hypovolemic shock after the loss of large amounts of blood and plasma. Following physical assessment, which findings lead the nurse to believe the client's body is compensating for this fluid loss? Select all that apply. A)Increased heart rate B)Vasodilation with warm extremities C)Diuresiswith output of 100 mL/hour D)The client complaining of extreme thirst E)Deep, rapid respirations

A)Increased heart rate D)The client complaining of extreme thirst E)Deep, rapid respirations Compensatory mechanisms in hypovolemic shock include increased heart rate, peripheral vasoconstriction, and fluid and sodium retention in order to preserve vascular volume. Urine output decreases very quickly in hypovolemic shock. Thirst is an early symptom in hypovolemic shock. As shock progresses, the respirations become rapid and deep to compensate for the increased production of acid and decreased availability of oxygen

A 2-year-old child is admitted to pediatric unit with bronchiolitis. The nurse calls the physician fearing the child is going into respiratory failure based on which of the following assessment findings? Select all that apply. A)Increased respiratory rate to 44 breaths/minute B)Substernal retractions becoming more pronounced C)New-onset expiratory grunting D)Productive cough with white secretions E)Faint wheezes noted in the posterior lung base

A)Increased respiratory rate to 44 breaths/minute B)Substernal retractions becoming more pronounced C)New-onset expiratory grunting Children with impending respiratory failure due to airway or lung disease have rapid breathing; exaggerated use of the accessory muscles; retractions, which are more pronounced in the child than in the adult because of higher chest compliance; nasal flaring; and grunting during expiration

An 86-year-old male client is disappointed to learn that he has class II heart failure despite a lifelong commitment to exercise and healthy eating. Which of the following age-related changes predisposes older adults to developing heart failure? A)Increased vascular stiffness B)Orthostatic hypotension C)Increased cardiac contractility D)Loss of action potential

A)Increased vascular stiffness Increased vascular stiffness in older adults causes a progressive increase in systolic blood pressure with advancing age, which in turn contributes to the development of left ventricular hypertrophy and altered diastolic filling. A loss of action potential does not typically accompany aging, and contractility tends to decrease as a result of cardiac stiffness. Orthostatic hypotension is neither a normal age-related change nor a cause of heart failure.

An elderly female client who reports increasing fatigue has been diagnosed with aortic stenosis, a disease that her primary care provider believes may have been long-standing. Which of the following compensatory mechanisms has most likely maintained the woman's ejection fraction until recently? A)Left ventricular hypertrophy B)Increased blood pressure C)Increased heart rate and stroke volume D)Aortic dilation

A)Left ventricular hypertrophy Because aortic stenosisdevelops gradually, the left ventricle has time to adapt by increasing in wall thickness to maintain a normal ejection fraction. Increased blood pressure and heart rate and dilation of the aorta are not responses that mitigate the effects or aortic stenosis.

Which of the following diagnostic/assessment findings would been seen in a client with worsening mitral valve stenosis? Select all that apply. A)Low-pitched diastolic murmur that is increasing in duration B)Sharp elevation in left atrial pressure C)Decreased cardiac output D)Severe elevation in left ventricular end-diastolic pressure E)Left ventricle increases its stroke volume

A)Low-pitched diastolic murmur that is increasing in duration B)Sharp elevation in left atrial pressure C)Decreased cardiac output The increased left atrial pressure eventually is transmitted to the pulmonary venous system, causing pulmonary congestion. A characteristic auscultatory finding in mitral stenosis is an opening snap following the second heart sound, which is caused by the stiff mitral valve. As the stenosis worsens, there is a localized low-pitched diastolic murmur that increases in duration with the severity of the stenosis. Manifestations are related to the elevation in left atrial pressure and pulmonary congestion such as dyspnea with exertion, decreased cardiac output owing to impaired left ventricular filling, and left atrial enlargement with the development of atrial arrhythmias and mural thrombi. Severe elevation in left ventricular end-diastolic pressure and left ventricle increases its stroke volume occur with aortic regurgitation.

Most respiratory disorders in infants produce a decrease in: A)Lung compliance B)Airway resistance C)Oxygen demand D)Respiratory rate

A)Lung compliance Most respiratory disorders in infants produce a decrease in lung compliance or an increase airway resistance. Respiratory disorders increase the cellular demand for oxygen because the alveolar-capillary membrane is often impaired by the disease process. A compensatory tachypnea is an attempt to deliver more oxygenated blood to hypoxic lung tissue

One of the principal mechanisms by which the heart compensates for increased workload is: A)Myocardial hypertrophy B)Sodium and water retention C)Endothelin vasoconstrictors D)Ventricular wall tension increase

A)Myocardial hypertrophy The development of myocardial hypertrophy constitutes one of the principal mechanisms by which the heart compensates for an increase in workload. There are at least two types of endothelin receptors, and it is thought that the peptide may play a role in mediating noncompensatory pulmonary hypertension in persons with heart failure. One effect of a lowered cardiac output in heart failure is a noncompensatory reduction in renal blood flow and glomerular filtration rate, which leads to salt and water retention. Because increased wall tension increases myocardial oxygen requirements, it can produce noncompensatory ischemia and further impairment of cardiac function.

While lecturing on blood pressure, the nurse will emphasize that the body maintains its blood pressure by adjusting the cardiac output to compensate for changes in which of the following physiologic processes? A)Peripheral vascular resistance B)Electrical impulses in the heart C)Release of stress hormones D)Rigidity of the ventricular walls

A)Peripheral vascular resistance The systolic and diastoliccomponents of blood pressure are determined by cardiac output and total peripheral vascular resistance and can be expressed as the product of the two (blood pressure = cardiac output × total peripheral resistance). The body maintains its blood pressure by adjusting the cardiac output to compensate for changes in peripheral vascular resistance, and it changes the peripheral vascular resistance to compensate for changes in cardiac output. Electrical impulses from the SA node regulate heart rate. Release of stress hormones and rigidity of the ventricular walls do not primarily influence BP; however, they may impact this secondarily

Which of the following is most likely to precipitate an asthmatic attack in a child with a diagnosis of extrinsic, or atopic, asthma? A)Pet dander B)Cold weather C)Stress D)Respiratory tract infections

A)Pet dander Extrinsic or atopicasthma is typically initiated by a type I hypersensitivity reaction induced by exposure to an extrinsic antigen or allergen such as pet dander. Intrinsic or nonatopic asthma triggers include respiratory tract infections, exercise, hyperventilation, cold air, drugs and chemicals, hormonal changes and emotional upsets, airborne pollutants, and gastroesophageal reflux.

A client with a history of heart failure and COPD (caused by 60 pack/year smoking) presents to the clinic with the following complaints: auscultation of breath sounds reveal absent/diminished breath sounds in the right lower lobe. Which other manifestations lead the health care provider to suspect the client may have developed atelectasis? Select all that apply. A)Respiratory rate—32; pulse rate—122 beats/minute. B)"Having a hard time catching my breath." C)"Seems like I'm not making much water (decreased urine production)." D)Using accessory muscles to help him breathe. E)Copious amounts of thick, green sputum.

A)Respiratory rate—32; pulse rate—122 beats/minute. B)"Having a hard time catching my breath." D)Using accessory muscles to help him breathe. Atelectasis is caused most commonly by airway obstruction rather than a vascular obstruction. The clinical manifestations of atelectasisinclude tachypnea (respiratory rate of 32), tachycardia (pulse rate of 122) dyspnea (hard time catching breath), cyanosis, signs of hypoxemia, diminished chest expansion, absence of breath sounds, and intercostal retractions (use of accessory muscles). Both chest expansion and breath sounds are decreased on the affected side. There may be intercostal retraction (pulling in of the intercostal spaces) over the involved area during inspiration. Urine production is not related to atelectasis. Copious green sputum is associated with infection.

Assessment of an elderly female client reveals the presence of bilateral pitting edema of the client's feet and ankles and pedal pulses that are difficult to palpate. Auscultation of the client's lungs reveals clear air entry to bases, and the client's oxygen saturation level is 93%, and vital signs are within reference ranges. What is this client's most likely health problem? A)Right-sided heart failure B)Pericarditis C)Cardiogenic shock D)Cor pulmonale

A)Right-sided heart failure A major effect of right-sided heart failure is the development of peripheral edema. A client who is in shock would not have stable vital signs. Cor pulmonale would be accompanied by manifestations of lung disease. Pericarditis is an inflammation of the pericardium exhibited by fever, precordial pain, dyspnea, and palpitations.

Which of the following would be considered a clinical manifestation of acute pericarditis? Select all that apply. A)Sharp, abrupt onset of chest pain that radiates to the neck B)Pericardial friction rub C)Narrowed pulse pressure D)Muffled heart sounds E)Abnormal ECG results

A)Sharp, abrupt onset of chest pain that radiates to the neck B)Pericardial friction rub E)Abnormal ECG results The manifestations of acute pericarditis include a triad of chest pain, an auscultatory pericardial friction rub, and electrocardiographic (ECG) changes. The pain usually is sharp and abrupt in onset, occurring in the precordial area, and may radiate to the neck, back, abdomen, or side. Pain in the scapular area may result from irritation of the phrenic nerve. The pain typically is pleuritic (aggravated by inspiration and coughing) and positional (decreases with sitting and leaning forward) because of changes in venous return and cardiac filling. A pericardial friction rub results from the rubbing and friction between the inflamed pericardial surfaces. Persons with cardiac tamponade usually have heart sounds that become muffled because of the insulating effects of the pericardial fluid and reduced cardiac function. A key diagnostic finding in cardiac tamponade is pulsus paradoxus or an exaggeration of the normal variation in the systolic blood pressure.

A client with an 80-pack-year history of tobacco smoking has presented to the clinic complaining of "bronchitis" cough for the past 5 months, weight loss, and shortness of breath. Today, this client "got scared" when he coughed up blood in his sputum. The health care provider is concerned this client may have which of the following possible diagnoses? A)Small cell lung cancer due to smoking history B)Tuberculosis due to long period of coughing C)Pulmonary embolism due to blood in sputum D)Pneumothoraxrelated to chronic lung infection weakening the alveoli

A)Small cell lung cancer due to smoking history Small cell lung cancer has the strongest association with cigarette smoking and is rarely observed in someone who has not smoked; brain metastasis is common. The earliest symptoms (of lung cancer) usually are chronic cough, shortness of breath, and wheezing because of airway irritation and obstruction. Hemoptysis (i.e., blood in the sputum) occurs when the lesion erodes into blood vessels. There is no indication the client has risk factors for TB. Pulmonary emboli result from blood clots traveling to the lungs. Pneumothorax would cause different symptoms and be an acute, abrupt onset

The plaques in a client's coronary arteries are plentiful, and most have small- to moderate-sized lipid cores with thick fibrous caps. This form of atherosclerosis is most closely associated with which of the following diagnoses? A)Stable angina B)Non-ST-segment elevation MI C)ST-segment elevation MI D)Unstable angina

A)Stable angina The fixed or stable plaque is commonly associated with stable angina, and the unstable plaque is implicated in unstable angina and myocardial infarction (MI).

A 6-month-old infant has been hospitalized with acute bronchiolitis. Which of the following treatments should be prioritized in the infant's care? A)Supplementary oxygen therapy B)Intravenous antibiotics C)Transfusionof fresh frozen plasma D)Tracheotomy

A)Supplementary oxygen therapy Bronchiolitis necessitates supplementary oxygen therapy. Antibiotics are ineffective due to the viral etiology. Recovery usually occurs within several days, and tracheotomy is necessary only in the event of severe complications. Plasma transfusion is not a relevant treatment modality.

A client with newly diagnosed squamous cell carcinoma of the lung asks, "So how do we treat this cancer?" Which response from the health care provider is most accurate? Select all that apply. A)Surgery to remove tumor B)Radiation therapy C)Chemotherapy D)Stem cell transplant E)Monoclonal antibody

A)Surgery to remove tumor B)Radiation therapy C)Chemotherapy Although all forms of lung cancer are serious and potentially fatal diagnoses, individuals with small cell lung cancer, adenocarcinoma, and large cell lung cancer often face prognoses that are worse than those associated with squamous cell lung cancer. Treatment methods for NSCLC include surgery, radiation therapy, and systemic chemotherapy. These treatments may be used singly or in combination. Stem cell transplant is not utilized in lung cancer. New trials are exploring the use of monoclonal antibodies but have not been approved by the FDA at this time.

A client is admitted to hospital to rule out Legionnaire disease following a canoe trip where he was sprayed in the face with a lot of "creek" water. Which of the following manifestations are characteristic of Legionnaire pneumonia? Select all that apply. A)Temperature of 103.5°, pulse 80 B)"Talking but not making a lot of sense" (confusion) C)Decreased abdominal bowel sounds D)Productive cough with thick, yellow secretions E)Chest x-ray that reveals areas of consolidation suggestive of pneumonia

A)Temperature of 103.5°, pulse 80 B)"Talking but not making a lot of sense" (confusion) E)Chest x-ray that reveals areas of consolidation suggestive of pneumonia Legionellapneumonia typically presents acutely with malaise, weakness, lethargy, fever, and dry cough. Other manifestations include disturbances of central nervous system function and elevation in body temperature, sometimes to more than 104°F (40°C). The presence of pneumonia along with diarrhea, hyponatremia, and confusion is characteristic of Legionella pneumonia. The disease causes consolidation of lung tissues and impairs gas exchange. Another characteristic of the disease is a lack of a normal pulse-temperature relationship in which a fever is not accompanied by an appropriate rise in heart rate. For example, a temperature of 102°F is normally accompanied by a heart rate of 110 beats/minute; in Legionella pneumonia it is often less than 100 beats/minute.

A nurse who provides weekly care in a homeless shelter has unknowingly inhaled airborne Mycobacteriumtuberculosis and has subsequently developed latent tuberculosis infection. Which of thefollowing is true of this nurse? A)The nurse is likely asymptomatic. B)The nurse is now immune to more severe tuberculosis infection. C)The nurse can spread tuberculosis to others. D)The nurse has active tuberculosis infection.

A)The nurse is likely asymptomatic. Latent TB infection is not an active form of TB, and affected individuals are asymptomatic and cannot pass on the disease to others. It does not confer immunity

An elderly client who has been restricted to bed by numerous comorbidities for several weeks has been diagnosed with a large pleural effusion. Which of the following treatment modalities is most likely to resolve the client's most recent health problem?A)Thoracentesis B)Supplementary oxygen therapy C)Administration of corticosteroids D)Administration of bronchodilators

A)Thoracentesis With large effusions, thoracentesis may be used to remove fluid from the intrapleural space and allow for reexpansion of the lung.

An IV drug abuser has been diagnosed with infective endocarditis. He is in the emergency department reporting increasing shortness of breath, rapid breathing, chest pain that worsens with breathing, and coughing up blood. The health care provider recognizes this may be caused by: A)Vegetative emboli traveling in the blood stream to the lungs B)Blood clots in the left ventricle traveling through the aorta C)Microemboli being developed in the carotids by Staphylococcus epidermidis D)Infarctionof the tissue surrounding the endocardium of the heart

A)Vegetative emboli traveling in the blood stream to the lungs The client is exhibiting signs of pulmonary emboli. The infectious loci continuously release bacteria into the bloodstream and are a source of persistent bacteremia, sometimes contributing to pericarditis. As the lesions grow, they cause valve destruction and dysfunction such as regurgitation, ring abscesses with heart block, and perforation. The loose organization of these lesions permits the organisms and fragments of the lesions to form emboli and travel in the bloodstream, causing cerebral, systemic, or pulmonary emboli. If clots are in the left ventricle, they will travel to the brain or kidneys. If emboli are located in the carotids, they will travel to the brain tissue. Infarction of heart tissue will exhibit signs of a myocardial infarction, not pulmonary

While teaching a client with new-onset right-sided heart failure, the nurse should educate the client to monitor for fluid accumulation by: A)Weighing every day at the same time with same type of clothing B)Measuring all of the client's urine output daily to check for a decrease in output C)Listening to the breath sound with a stethoscope every morning D)Take blood pressure daily and call doctor if it is decreased

A)Weighing every day at the same time with same type of clothing When the right heart fails, a damming back of blood occurs, leading to its accumulation in the systemic venous system, causing an increase in right atrial, right ventricular end-diastolic, and systemic venous pressures. The accumulation of fluid (edema) is evidenced by a gain in weight (i.e., 1 pint of accumulated fluid results in a 1-pound weight gain). Shortness of breath due to congestion of the pulmonary circulation is one of the major manifestations of left-sided heart failure. It is unrealistic to expect clients to listen to their own breath sounds. BP measurement could be an intervention; however, it is not a primary indicator of edema from right-sided heart failure. With impairment of left heart function, there is a decrease in cardiac output, with resulting decreased renal perfusion and output.

Which of the following clients at the clinic should be encouraged to receive the pneumococcal polysaccharide vaccine (PPSV23)? A client: Select all that apply. A)Who is 65 years old with chronic asthma B)With a smoking history C)Who is a young adult with HIV-positive results D)Who is school-aged and has received a liver transplant E)Who is a teenager with history of kidney disease

A)Who is 65 years old with chronic asthma B)With a smoking history The PPSV23 vaccine consists of the 23 most common capsular serotypes that cause the most common invasive pneumococcal disease. It is recommended for all adults 65 years of age and older and for those 2 years of age and older who are at high risk for the disease. It is also recommended for adults who smoke or have asthma. The PCV13 protects against 13 types of pneumococcal bacteria. It is recommended for use in infants and young children and for all adults 50 years of age and older who have conditions that weaken the immune system such as HIV infection, organ transplantation, leukemia, lymphoma, and severe kidney disease.

A nurse is performing client health education with a 68-year-old man who has recently been diagnosed with heart failure. Which of the following statements demonstrates an accurate understanding of his new diagnosis? A)"I'll be sure to take my beta blocker whenever I feel short of breath." B)"I'm going to avoid as much physical activity as I can so that I preserve my strength." C)"I know it's healthy to drink a lot of water, and I'm going to make sure I do this from now on." D)"I'm trying to think of ways that I can cut down the amount of salt that I usually eat."

D)"I'm trying to think of ways that I can cut down the amount of salt that I usually eat." Salt and fluid restrictions are indicated for most clients with heart failure (HF). Beta blockers do not address shortness of breath, and cardiac medications are not normally taken in response to acute symptoms. Clients should be encouraged to maintain, and increase, physical activity within the limits of their condition.

A client awaiting a heart transplant is experiencing decompensation of her left ventricle that will not respond to medications. The physicians suggest placing the client on a ventricular assist device (VAD). The client asks what this equipment will do. The health care providers respond: A)"Pull your blood from the right side of the heart and run it through a machine to oxygenate it better, and then return it to your body." B)"Measure the pressures inside your heart continuously to asses pumping ability of your left ventricle." C)"Have a probe at the end of a catheter to obtain thermodilution measures, so cardiac output can be calculated." D)"This device will decrease the workload of the myocardium while maintaining cardiac output and systemic arterial pressure."

D)"This device will decrease the workload of the myocardium while maintaining cardiac output and systemic arterial pressure." Refractory heart failure reflects deterioration in cardiac function that is unresponsive to medical or surgical interventions. Ventricular assist devices (VADs) are mechanical pumps used to support ventricular function. VADs are used to decrease the workload of the myocardium while maintaining cardiac output and systemic arterial pressure. This decreases the workload on the ventricle and allows it to rest and recover. The rest of the distractors relate to the monitoring in an ICU of cardiac functioning. Invasive hemodynamic monitoring may be used for assessment in acute, life-threatening episodes of heart failure. With the balloon inflated, the catheter monitors pulmonary capillary pressures (i.e., pulmonary capillary wedge pressure or pulmonary artery occlusion pressure), which reflect pressures from the left ventricle. The pulmonary capillary pressures provide a means of assessing the pumping ability of the left ventricle. One type of pulmonary artery catheter is equipped with a thermistor probe to obtain thermodilution measurementsof cardiac output.

A health care provider was asked by a client, "Why do my hands turn blue when I drive my car in the winter without gloves?" Which of the following is the best response? A)"Nothing to worry about. We all develop this as we age." B)"We better order a CT scan. It might be due to a blood clot in your radial artery." C)"This sounds like an inflammation in the lining of your veins. You need to take some NSAIDs." D)"Your arteries in your hands/fingers are going into spasm, which decreases blood flow and circulating oxygen."

D)"Your arteries in your hands/fingers are going into spasm, which decreases blood flow and circulating oxygen." Raynaud disease is caused by vasospasms of small distal arteries; thromboangiitis obliteransis caused by an inflammatory process that affects veins and nerves.

Which of the following individuals is most clearly in need of diagnostic testing for lung cancer? A)A client who has required hospitalization with a fever and the production of copious lung secretions B)A client with a history of secondary tuberculosis who failed to complete his prescribed course of antibiotics C)A woman who complains of recurrent lower respiratory infections and who has sought care for increasing shortness of breath D)A man who demonstrates wasting of the pelvic and shoulder muscles combined with signs of hypercalcemia

D)A man who demonstrates wasting of the pelvic and shoulder muscles combined with signs of hypercalcemia Hypercalcemia and wasting of the proximal muscles of the pelvic and shoulder girdles are indicative of the paraneoplastic manifestations of lung cancer. The other cited respiratory conditions warrant follow-up and treatment but are not particularly suggestive of a neoplastic etiology.

A postsurgical client reports calf pain combined with the emergence of swelling and redness in the area, which have culminated in a diagnosis of deep vein thrombosis. What treatment options will be of greatest benefit to this client? A)Analgesics and use of a pneumatic compression device B)Massage followed by vascular surgery C)Frequent ambulation and the use of compression stockings D)Anticoagulation therapy and elevation of the leg

D)Anticoagulation therapy and elevation of the leg Anticoagulants, immobilization, and elevation of the affected extremity are used in the treatment of DVT. Interventions that are used to prevent DVT (ambulation; compression stockings; compression devices) may be harmful when a DVT is present. Surgery is not normally required, and massage has the potential to dislodge a DVT.

A car accident client is admitted with a chest tube following pneumothorax. He also has an elevated blood alcohol level. When the nurse enters his room, she notes the client is dyspneic, short of breath, and holding his chest tube in his hand. When the nurse pulls the linens back, she finds a "sucking" chest wound. After calling a "code blue," the next priority intervention would be to: A)Place the client's meal napkin over the wound B)Observe and wait for the code blue team to bring equipment C)Try to calm the patient down by maintaining therapeutic communication D)Apply a Vaseline gauze (airtight) dressing over the insertion site

D)Apply a Vaseline gauze (airtight) dressing over the insertion site The client has a medical emergency. Sucking chest wounds, which allow air to pass in and out of the chest cavity, should be treated by promptly covering the area with an airtight covering. Chest tubes are inserted as soon as possible. The other interventions will not help minimize the amount of air entering the pleural space.

Persistent cyanosis has led an infant's care team to suspect a congenital heart defect. Which of the following assessment findings would suggest coarctation of the infant's aorta? A)The child has a split S2 heart sound on auscultation. B)ECG reveals atrial fibrillation. C)The child experiences apneic spells after feeding. D)Blood pressure in the child's legs is lower than in the arms.

D)Blood pressure in the child's legs is lower than in the arms. The classic sign of coarctation of the aorta is a disparity in pulsations and blood pressures in the arms and legs. In coarctation, the pressure in the legs is lower and may be difficult to obtain. A split S2, dysrhythmias, and apneic spells are not characteristics of coarctation of the aorta.

Which elevated serum marker for systemic inflammation is now considered a major risk factor for atherosclerosis and vascular disease? A)Leukocytosis B)Homocysteine C)Serum lipoprotein D)C-reactive protein

D)C-reactive protein CRPis an acute-phase reactant synthesized in the liver that is a marker for systemic inflammation. A number of population-based studies have demonstrated that baseline CRP levels can predict future cardiovascular events among apparently healthy individuals.High-sensitivity CRP (hs-CRP) may be a better predictor of cardiovascular risk than lipid measurement alone. Homocysteine and serum lipoprotein are also serum markers, but they do not identify inflammation. Leukocytosis is an indicator of infection rather than inflammation alone.

The client is immobilized following a hip injury and has begun demonstrating lower leg discoloration with edema, pain, tenderness, and increased warmth in the midcalf area. He has many of the manifestations of: A)Stasis ulcerations B)Arterial insufficiency C)Primary varicose veins D)Deep vein thrombosis

D)Deep vein thrombosis Venous insufficiency with deep vein thrombus formation is characterized by discoloration, edema, pain, tenderness, and warmth most commonly in the mid- or lower calf area of the legs. Immobility raises the risk for thrombus formation. The skin is intact, so venous stasis ulcerations are not present. Distended torturous veins (varicosity manifestations) are not present.

While in the ICU, a client's status changes. The health care providers suspect heart failure. Which of the following diagnostic procedures would give the staff information about pulmonary capillary pressures, which will lead to the most appropriate interventions?A)Echocardiography B)Radionuclide ventriculography C)Cardiac magnetic resonance imaging D)Hemodynamic monitoring

D)Hemodynamic monitoring Invasive hemodynamicmonitoring may be used for assessment in acute, life-threatening episodes of heart failure. These monitoring methods include central venous pressure (CVP), pulmonary artery pressure monitoring, measurements of cardiac output, and intra-arterial measurements of blood pressure. Echocardiography plays a key role in assessing ejection fraction, right and left ventricular wall, wall thickness, ventricular chamber size, valve function, heart defects, and pericardial disease. Radionuclide ventriculography is recommended if there is reason to suspect coronary artery disease or ischemia as the underlying cause for heart failure. Cardiac magnetic resonance imaging and cardiac computed tomography are used to document ejection fraction, ventricular preload, and regional wall motion.

Which of the following manifestations typically accompanies an asthmatic attack? A)Decreased residual volume B)Decreased pulmonary arterial pressure C)Prolonged inspiration D)Hyperinflation of the lungs

D)Hyperinflation of the lungs During a prolonged attack, air becomes trapped behind the occluded and narrowed airways, causing hyperinflation of the lungs. This produces an increase in the residual volume of the lungs. Pulmonary arterial pressure tends to increase and expiration becomes prolonged

A client has just returned from his surgical procedure. During initial vital sign measurements, the nurse notes that the client's heart rate is 111 beats/minute and the BP is 100/78 (borderline low). In this early postoperative period, the nurse should be diligently monitoring the client for the development of: A)Pulmonary embolism due to development of deep vein thrombosis B)Side effects from versed administration causing excessive vasoconstriction C)Renal failure due to an overdose of medication D)Hypovolemic shock due to acute intravascularvolume loss

D)Hypovolemic shock due to acute intravascularvolume loss Hypovolemic shock is characterized by diminished blood volume such that there is inadequate filling of the vascular compartment. Hypovolemic shock also can result from an internal hemorrhage or from third-space losses, when extracellular fluid is shifted from the vascular compartment to the interstitial space or compartment, without fluid movement in/out of the cells. Within seconds after the onset of hemorrhage or the loss of blood volume, compensatory manifestations of tachycardia, vasoconstriction, and other signs of sympathetic and adrenal medullary activity appear. There is no indication that this client has developed a pulmonary embolism, is having side effects from versed administration, or is going into renal failure due to an overdose of medication.

A client withheart failure asks, "Why am I taking a 'water pill' when it's my heart that is having a problem?" While educating the client about the Frank-Starling mechanism, which of the following explanations is most appropriate to share? A)"You must be drinking way too many liquids. Your kidneys cannot filter all that you are drinking during the day." B)"Since your heart is not pumping efficiently, the kidneys are getting less blood flow; therefore, the kidneys are holding on to sodium and water." C)"Your heart muscle is overstretched, so it's not able to pump all the blood out. The prescribed 'water pills' help by decreasing your weight." D)"Since your heart function is impaired, the lungs are not able to oxygenate the blood and your kidneys are wearing out."

B)"Since your heart is not pumping efficiently, the kidneys are getting less blood flow; therefore, the kidneys are holding on to sodium and water." In heart failure with a reduced ejection fraction, a decrease in cardiac output and renalblood flow leads to increased sodium and water retention by the kidney with a resultant increase in vascular volume and venous return to the heart and an increase in ventricular end-diastolic volume. Drinking water may increase volume but is not the physiological reason for retention of fluid. Diuretics do decrease weight as a result of diuresis, but weight loss is not the purpose for giving diuretics. The lungs are not the primary cause of heart failure.

While administering a tuberculin (TB) skin test, a client who is HIV positive asks, "I heard from my friends, this test may not work on me since I have HIV." The health care provider's best response would be: A)"This test is 99.9% specific, so it will give us an accurate result." B)"Sometimes immunocompromisedclients will have negative results if you are unable to mount a normal immune reaction." C)"Most of the time, with HIVpositive clients, we see more false-positive results since you may have a similar infection in your body." D)"If your test comes backpositive, we will send a blood test off to a special lab to confirm you really have TB before we start treatment."

B)"Sometimes immunocompromisedclients will have negative results if you are unable to mount a normal immune reaction." Hypersensitivity response to the tuberculin test depends on cell-mediated immunity. Inherent anergy in persons with HIV infection and other immunocompromised states causes a false-negative test result, which can mean that the person has a true lack of exposure to tuberculosis or is unable to mount an immune response to the test. False-positive results can also occur—for example, in persons who have nontuberculosis mycobacterial infection or who have received the Bacillus Calmette-Guérin (BCG) vaccine for tuberculosis prevention.

An elderly client newly diagnosed with systolic hypertension asks her health care provider why this happens. The most accurate response would be: A)"Everyone over the age of 50 tends to have their blood pressure creep up over the years." B)"With age, your arteries lose their elasticity and are replaced with collagen, which makes your arteries stiffer." C)"Your heart has to work harder to pump blood through your vessels as you get older." D)"If you slow down and rest more, your blood pressures will more than likely return to its normal level."

B)"With age, your arteries lose their elasticity and are replaced with collagen, which makes your arteries stiffer." Systolic blood pressure rises almost linearly between 30 and 84 years of age, whereas diastolic pressure rises until 50 years of age and then levels off or decreases. This rise in systolic pressure is thought to be related to increased stiffness of the large arteries. With aging, the elastin fibers in the walls of the arteries are gradually replaced by collagen fibers that render the vessels stiffer and less compliant.

A client asks, "Why do I have clogged arteries but my neighbor has higher 'bad cholesterol' levels and yet he is just fine?" The health care provider bases the reply on which of the following physiological principles about lipoprotein? A)"Your neighbor probably has higher amounts of good cholesterol (HDL) as well." B)"You more than likely have small, dense type of 'bad cholesterol' (LDL)." C)"Your neighbor has larger 'bad cholesterol' particles that can move into blood vessels but park in joints/tendons." D)"You must have a genetic predisposition to having clogged arteries."

B)"You more than likely have small, dense type of 'bad cholesterol' (LDL)." There are different types of LDL, and some people with markedly elevated LDL do not develop atherosclerotic vascular disease, whereas other people with only modest elevations in LDL develop severe disease. Small, dense LDL is more toxic or atherogenic to the endothelium than large, buoyant LDL. It is more likely to enter the vessel wall, become oxidized, and trigger the atherosclerotic process.

Which of the following individuals is suffering the effects of acute coronary syndrome (ACS)? A)A client whose most recent ECG indicates that silent myocardial ischemia has occurred B)A client who occasionally experiences persistent and severe chest pain when at rest C)A client who sometimes experiences chest pain when climbing stairs D)A client who has recently been diagnosed with variant (vasospastic) angina

B)A client who occasionally experiences persistent and severe chest pain when at rest The onset of STEMI involves abrupt and significant chest pain. The pain typically is severe, often described as being constricting, suffocating, and crushing. Substernal pain that radiates to the left arm, neck, or jaw is common, although it may be experienced in other areas of the chest and back. Unlike that of angina, the pain associated with MI is more prolonged and not relieved by rest or nitroglycerin. Silent MI, stable angina, and variant, or vasospastic, angina are subtypes of chronic ischemic coronary artery disease

Which of the following clients would be considered at high risk for developing pneumonia (both community and hospital setting)? Select all that apply. A)A teenager who spends a lot of time at local coffee shopsusing Wi-Fi to chat with friends B)A young adult in motorcycle accident with head injury requiring tracheostomy and mechanical ventilation C)A college female who is pregnant (unplanned) who has been consuming alcohol prior to positive pregnancy test D)A HIV-positive client with a WBC count of 2000 who has been camping near a commercial farm raising chickens for food E)A school-aged child with severe asthma controlled by steroids admitted for an exacerbation

B)A young adult in motorcycle accident with head injury requiring tracheostomy and mechanical ventilation D)A HIV-positive client with a WBC count of 2000 who has been camping near a commercial farm raising chickens for food E)A school-aged child with severe asthma controlled by steroids admitted for an exacerbation Persons requiring intubation and mechanical ventilation are particularly at risk, as are those with compromised immune function, chronic lung disease (like asthma), and airway instrumentation, such as endotracheal intubation or tracheotomy. Ventilator-associated pneumonia is pneumonia that develops in mechanically ventilated clients more than 48 hours after intubation. Neutropenia and impaired granulocyte function predispose to infections caused by S. aureus, Aspergillus, gram-negative bacilli, and Candida. Pneumonia in immunocompromised persons remains a major source of morbidity and mortality. The epithelial cells of critically and chronically ill persons are more receptive to binding microorganisms that cause pneumonia.

A distinguishing feature of viral influenza is: A)Direct contact transmission B)Abrupt-onset, profound malaise C)Constant pounding headache D)Profuse watery nasal discharge

B)Abrupt-onset, profound malaise One distinguishing feature of influenza is the rapid onset, sometimes within minutes, of profound malaise. As with many viral respiratory tract infections, transmission is by aerosol or direct contact. In the early stages, the symptoms of influenza often are indistinguishable from other viral infections—fever, chills, malaise, muscle aching, headache, profuse watery nasal discharge, nonproductive cough, and sore throat. Persons with chronic rhinosinusitis complain of a constant pounding headache.

In addition to direct invasion of the vascular wall by an infectious agent, this pathogenic mechanism is a common cause of vasculitis?A)Necrotizing granulomatous B)Tissue necrosis C)Mononuclear cells D)Immune-mediated inflammation

D)Immune-mediated inflammation The two most common pathogenic mechanisms of vasculitisare direct invasion of the vascular wall by an infectious agent and immune-mediated inflammation. The most common mechanisms that initiate noninfectious vasculitis are pathological immune responses that result in endothelial activation, with subsequent vessel obstruction, and ischemia of the dependent tissue. In almost all forms of vasculitis, the triggering event initiating and driving the inflammatory process is unknown. Medium-size vessel vasculitides produce necrotic tissue damage. Large-vessel vasculitides involve mononuclear cells. Wegener granulomatosis is characterized by a triad of acute necrotizing granulomatous lesions of the upper respiratory tract (ear, nose, sinuses, and throat), necrotizing vasculitis of the affected small- to medium-sized vessels of the lungs and respiratory airways, and renal disease in the form of focal necrotizing glomerulonephritis.

A client with a history of emphysema is experiencing hypoxemia after a taxing physical therapy appointment. Which of the following physiologic phenomena will occur as a consequence of hypoxemia?A)Peripheral vasodilation B)Necrosis C)Hypoventilation D)Increased heart rate

D)Increased heart rate Consequences of hypoxemia include peripheral vasoconstriction, hyperventilation, and increased heart rate. Mild to moderate hypoxemia does not result in cell death and necrosis.

A client has presented to an ambulatory clinic complaining of a persistent headache. What assessments should the clinician conduct to differentiate between rhinosinusitis and alternative health problems? A)Take a sputum sample for culture and sensitivity. B)Compare the client's oral, tympanic, and axillary temperatures and order a white blood cell count. C)Palpate the client's lymph nodes and inspect the ears with an otoscope. D)Perform transillumination and ask the client if bending forward exacerbates the headache.

D)Perform transillumination and ask the client if bending forward exacerbates the headache. Transillumination is performed to confirm or rule out sinusitis, and sinusitis headache usually is exaggerated by bending forward, coughing, or sneezing. The other cited assessments do not differentiate between rhinosinusitis and other health problems.

A client with a diagnosis of chronic renal failure secondary todiabetes has seen a gradual increase in her blood pressure over the past several months, culminating in a diagnosis of secondary hypertension. Which of the following has most likely resulted in the client's increased blood pressure? A)Increased levels of adrenocortical hormones B)Activation of the renin-angiotensin-aldosterone mechanism C)Increased sympathetic stimulation by the autonomic nervous system (ANS) D)Coarctation of the client's aorta

B)Activation of the renin-angiotensin-aldosterone mechanism Renovascularhypertension refers to hypertension caused by reduced renal blood flow and activation of the renin-angiotensin-aldosterone mechanism. It is the most common cause of secondary hypertension, accounting for 1% to 2% of all cases of hypertension. The reduced renal blood flow that occurs with renovascular disease causes the affected kidney to release excessive amounts of renin, increasing circulating levels of angiotensin II. Angiotensin II, in turn, acts as a vasoconstrictor to increase peripheral vascular resistance and as a stimulus for increased aldosterone levels and sodium retention by the kidney. One or both of the kidneys may be affected. A renal etiology is not associated with secondary hypertension due to hormonal factors, sympathetic stimulation, or coarctation of the aorta.

On the 3rd day following an acute myocardial infarction, the client is being discharged home. The nurse is explaining how the heart tissue heals following an MI. "Since today is your 3rd day after your heart attack, the tissue is: A)Soft, mushy, and yellow." B)Acutely inflamed." C)Forming granulation tissue." D)Developed a fibrous scar."

B)Acutely inflamed." Approximately 2 to 3 days post-myocardial infarction, an acute inflammatory response develops in the area surrounding the necrotic tissue. The damaged area is gradually replaced with vascularized granulation tissue, which in turn becomes less vascular and more fibrous in composition. At 4 to 7 days, the center of the infarcted area is soft and yellow. By the 7th week, the necrotic area is completely replaced by fibrous scar tissue.

Atherosclerotic peripheral vascular disease is symptomatic with at least 50% occlusion. The primary peripheral symptom, due to ischemia, is: A)Edema B)Calf pain C)Varicosities D)Strong pulse

B)Calf pain The primary symptom of chronic obstructive arterial disease is pain with walking or claudication. Typically, persons with the disorder complain of calf pain because the gastrocnemius muscle has the highest oxygen consumption of any muscle group in the leg during walking. The extremity will be thin, dry (no edema), and have weak low-pressure pulses due to severely reduced blood flow to the distal vessels.

A 25-year-old cystic fibrosis client presents to the clinic in obvious respiratory distress. Following physical exam, the health care provider suspects bronchiectasis based on which of the following findings? Select all that apply. A)Crushing, substernal chest pain B)Copious amounts of foul-smelling purulent sputum C)Neck vein distention D)Blood-tinged sputum E)Wheezing throughout the lung fields

B)Copious amounts of foul-smelling purulent sputum D)Blood-tinged sputum E)Wheezing throughout the lung fields Bronchiectasis is usually manifested by a chronic productive cough, often with several 100 mL of foul-smelling, purulent sputum a day. Hemoptysis is common. Dyspnea and wheezing occur in about 75% of clients. Crushing substernal chest pain and next vein distention are more suggestive of pulmonary emboli or myocardial infarction with right-sided heart failure.

A client consistently has an elevated systolic BP greater than 150mm Hg but a diastolic pressure in the 80s. The health care provider should be assessing for which of the following complications?A)Peripheral edema in lower legs from renal disease B)Crackles in bilateral lung bases caused by left-sided heart failure C)Ascites due to liver damage D)Confusion due to atherosclerosis of the carotid arteries

B)Crackles in bilateral lung bases caused by left-sided heart failure Systolic hypertension has been defined as a systolic pressure of 140 mm Hg or greater and a diastolic pressure of less than 90 mm Hg. Elevated pressures during systole favor the development of left ventricular hypertrophy, increased myocardial oxygen demands, and eventual left heart failure. Peripheral edema and ascites are usually associated with right-sided failure. Confusion due to blockages in carotid arteries can occur independent of hypertension.

For which of the following types of shock might intravenous antibiotic therapy be indicated? A)Obstructive shock B)Distributive shock C)Cardiogenic shock D)Hypovolemic shock

B)Distributive shock Septic shock is a subtype of distributive shock. The treatment of sepsis and septic shock focuses on control of the causative agent and support of the circulation and the failing organ systems. The administration of antibiotics that are specific for the infectious agent is essential. Swift and aggressive fluid administration is needed to compensate for third spacing, though which type of fluid is optimal remains controversial. Equally, aggressive use of vasopressor agents, such as norepinephrine or epinephrine, is needed to counteract the vasodilation caused by inflammatory mediators.

Clients with ischemic coronary vessel disease and acute coronary syndrome (ACS) are classified as low or high risk for acute myocardial infarction based on characteristics that include significant: A)Heart murmurs B)ECG changes C)Pulmonary disease D)Pericardial effusion

B)ECG changes Persons with ischemic coronary vessel disease and ACS are routinely classified as low or high risk for acute myocardial infarction based on clinical history, ECG variables, and serum cardiac biomarkers. Chronic pulmonary disease increases pulmonary vascular resistance, leading to right or left heart failure. Pericardial effusion increases intracardiac pressure and venous pressure. Heart murmurs result from turbulent blood flow through a diseased valve.

When evaluating the pulmonary function test results for a COPD client, which one correlates to the mismatch of ventilation and perfusion associated with this diagnosis? A)Forced vital capacity (FVC) is elevated. B)Forced expiratory volume (FEV) is decreased. C)Total lung capacity (TLC) is decreased. D)Marked decrease in residual volume (RV).

B)Forced expiratory volume (FEV) is decreased. In clients with chronic lung disease, the FVC is decreased, the FEV1.0is decreased, and the ratio of FEV1.0 to FVC is decreased. Lung volume measurements reveal a marked increase in RV, an increase in TLC, and elevation of the RV-to-TLC ratio.

A family brings their elderlyfather to emergency department. He has been exposed to pneumococcal pneumonia at his retirement home. Today, they noted a change in his mental status. They thought he might need some oxygen. Which of the other assessment findings would correlate with thisdiagnosis? Select all that apply. A)Expiratory wheezes throughout all lung fields B)Increase in chest pain with deep inspiration C)Absent breathe sounds on the entire right side of the lung D)Loss of appetite for past few days E)Purulent sputum with bloody patches

B)Increase in chest pain with deep inspiration D)Loss of appetite for past few days E)Purulent sputum with bloody patches During the initial stage, coughing brings up watery sputum and breath sounds are limited, with fine crackles. As the disease progresses, the character of the sputum changes; it may be blood tinged or rust colored to purulent. Pleuritic pain, a sharp pain that is more severe with respiratory movements, is common. Elderly persons are less likely to experience marked elevations in temperature; in these persons, the only sign of pneumonia may be a loss of appetite and deterioration in mental status. Expiratory wheezes are usually associated with asthma. Absent breath sounds on an entire lung field usually is associated with pneumothorax or respiratory failure

Atherosclerotic plaque is most likely to be unstable and vulnerable to rupture when the plaque has a thin fibrous cap over a: A)Red thrombus B)Large lipid core C)Calcified lesion D)Vessel wall injury

B)Large lipid core Plaque disruption causes thrombus formation, with white platelet-containing thrombi being associated with unstable angina. The major determinants of plaque vulnerability to disruption include the size of its lipid-rich core, lack of stabilizing smooth muscle cells, presence of inflammation with plaque degradation, and stability and thickness of its fibrous cap. Plaques with a thin fibrous cap overlaying a large lipid core are at high risk for rupture. Plaque tends to be stable or fixed unevenly in any area of the coronary arteries. Calcified plaque tends to be stable and encased in a thrombus, until it begins to degrade.

A client has arrived in the emergency department in cardiogenic shock. Which of the following assessment findings confirm this diagnosis? Select all that apply. A)Bright red color noted in the nail beds and lips B)Less than 5 mL dark, concentrated urine in the past hour C)BP reading of 80/65 D)Difficult to arouse with changes in level of consciousness E)Diminished breath sounds in the bases, bilaterally

B)Less than 5 mL dark, concentrated urine in the past hour C)BP reading of 80/65 D)Difficult to arouse with changes in level of consciousness The signs and symptoms of cardiogenicshock are consistent with those of end-stage heart failure. The lips, nail beds, and skin may become cyanotic because of stagnation of blood flow. Mean arterial and systolic blood pressures decrease due to poor stroke volume, and there is a narrow pulse pressure because of arterial vasoconstriction. Urine output decreases because of lower renal perfusion pressures and the increased release of aldosterone. Neurologic changes, such as alterations in cognition or consciousness, may occur because of low cardiac output and poor cerebral perfusion

Pleuritic chest pain associated with respiratory movements is usually described as: A)Bilateral B)Localized C)Continuous D)Substernal

B)Localized Pleuritisis usually unilateral and tends to be localized to the lower and lateral part of the chest; pain worsens with chest movements, such as deep breathing and coughing that accentuate pressure changes in the pleural cavity and increase movement of the inflamed or injured pleural surfaces. Musculoskeletal pain usually is bilateral and may occur as the result of frequent, forceful coughing. The pain associated with irritation of the bronchi usually is substernal and dull. Myocardial pain usually is located in the substernal area and is not affected by respiratory movements

A motor vehicle accident has resulted in critical injury for the driver of the car. The driver has hit the steering wheel with his chest and fractured his sternum and some ribs. Which of the following manifestations would lead the staff to suspect the driver has developed a tension pneumothorax? Select all that apply. A)Audible friction rub over the affected lung B)Mediastinal shift of the trachea toward one side C)Marked peripheral edema in lower limbs and ascites D)Atrial fibrillation noted on ECG printout E)Subcutaneous emphysema palpated in the upper chest/neck region

B)Mediastinal shift of the trachea toward one side E)Subcutaneous emphysema palpated in the upper chest/neck region With tension pneumothorax, the structures in the mediastinalspace shift toward the opposite side of the chest. There may be distention of the neck veins, subcutaneous emphysema (i.e., presence of air in the subcutaneous tissues of the chest and neck), and clinical signs of shock. Pneumothoraces do not typically cause a friction rub or dysrhythmias. Marked peripheral edema is associated with right-sided heart failure.

A teenaged cystic fibrosis client presents to the clinic. The health care provider (HCP) knows that cystic fibrosis (CF) causes severe chronic respiratory disease in children. In addition, the HCP should also focus his or her assessment on which of the other body systems affected by CF? A)Renal B)Pancreatic C)Cardiac D)Central nervous system

B)Pancreatic Cystic fibrosis (CF) is manifested by pancreatic exocrine deficiency and elevation of sodium chloride in the sweat. Cystic fibrosis (CF) is an inherited disorder involving fluid secretion by the exocrine glands in the epithelial lining of the respiratory, gastrointestinal, and reproductive tracts. Excessive loss of sodium in the sweat predisposes young children to salt depletion episodes. Respiratory manifestations are caused by an accumulation of viscid mucus in the bronchi, impaired mucociliary clearance, lung infections, bronchiectasis, and dilatation. The renal, cardiac, and CNS are usually not involved with CF manifestation.

A client with a history of disabling claudication now is in the emergency department with a lower limb that is turning dark purple to black associated with faint Doppler pedal pulses. The client will more than likely undergo: A)Surgery to remove the saphenous vein B)Percutaneous transluminal angioplasty and stent placement C)Injection of a potent anticoagulant into lower leg veins D)Whirlpool therapy with tight wrapping of lower legs immediately following

B)Percutaneous transluminal angioplasty and stent placement Treatment includes measures directed at protection of the affected tissues and preservation of functional capacity. Percutaneous or surgical intervention is typically reserved for the client with disabling claudication or limb-threatening ischemia. Surgery (i.e., femoropopliteal bypass grafting using a section of the saphenous vein) may be indicated in severe cases. Percutaneous transluminal angioplasty and stent placement, in which a balloon catheter is inserted into the area of stenosis and the balloon inflated to increase vessel diameter, is another form of treatment.

A family member comes rushing out of a client's room telling the nurse that theloved one can't breathe. The nurse has just left the room after hanging IV penicillin. Which of the following clinical manifestations lead the nurse to suspect the client is experiencing anaphylactic shock? Select all that apply. A)Incontinent of urine B)Severe bronchospasm C)Wheezing sound on inspiration D)Hives over entire body E)Swelling around the lips and eyes

B)Severe bronchospasm C)Wheezing sound on inspiration D)Hives over entire body E)Swelling around the lips and eyes Anaphylactic shock results from an immunologically mediated reaction in which vasodilator substances such as histamine are released into the blood. These substances cause vasodilation of arterioles and venules along with a marked increase in capillary permeability. The vascular response in anaphylaxis is often accompanied by life-threatening laryngeal edema and bronchospasm, circulatory collapse, contraction of gastrointestinal and uterine smooth muscle, and urticaria (hives) or angioedema. The onset and severity of anaphylaxis depend on the sensitivity of the person and the rate and quantity of antigen exposure. Signs and symptoms associated with impending anaphylactic shock include abdominal cramps; apprehension; warm or burning sensation of the skin, itching, and urticaria (i.e., hives); and respiratory distress such as coughing, choking, wheezing, chest tightness, and difficulty in breathing

Which of the following characteristics of the lungs of infants and small children creates an increased risk of respiratory disorders?A)Type II alveoli in children may overproduce surfactant. B)Smaller airways create a susceptibility to changes in airway resistance and airflow. C)The pneumotaxic center in the pons is underdeveloped until 8 years of age. D)There are fewer chemoreceptorsin the young medulla.

B)Smaller airways create a susceptibility to changes in airway resistance and airflow. Because the resistance to airflow is inversely related to the fourth power of the radius (resistance = 1/radius), relatively small amounts of mucus secretion, edema, or airway constriction can produce marked changes in airway resistance and airflow. Surfactant production is low early in life, and the respiratory center and chemoreceptors are present and functional in infants and children.

Which of the following medications helps treat the inflammatory reaction of asthma clients diagnosed with late-phase asthma response? A)Anticholinergic agents B)Systemic corticosteroids C)Long-acting β2-agonists D)Phosphodiesterase inhibitors

B)Systemic corticosteroids A short course of systemic corticosteroids, administered orally or parenterally, may be used for treating the inflammatory reaction associated with the late-phase response. The anticholinergic agents block cholinergic receptors and reduce intrinsic vagal tone that causes bronchoconstriction. The long-acting β2 agonists, available for administration by the inhaled or oral routes, act by relaxing bronchial smooth muscle. Theophylline, a phosphodiesterase inhibitor, is a bronchodilator that acts by relaxing bronchial smooth muscle

When lecturing to a group of students about the pathophysiological principles behind heart failure, the instructor explains that cardiac output represents: A)Strength of the right ventricular pump to move blood B)The amount of blood the heart pumps each minute C)The amount of blood pumped out of the heart with each beat D)The volume of blood stretching the heart muscle at the end of diastole

B)The amount of blood the heart pumps each minute Cardiac output, which is the major determinant of cardiac performance, reflects how often the heart beats each minute (heart rate) and how much blood it ejects with each beat (stroke volume). Preload reflects the volume of blood that stretches the ventricle at the end of diastole, just before the onset of systole.

The most important complication of atherosclerosis that may cause occlusion of small heart vessels is: A)Ulceration B)Thrombosis C)Fatty streaks D)Fibrous plaque

B)Thrombosis Thrombus formations on complicated atherosclerotic lesions are the result of sluggish blood flow and turbulence in the ulcerated plaque region. Fatty streaks are preatherosclerotic plaque changes in vessels. Fibrous plaque is part of the atherosclerosis formation, not a complication of it

A man sustained a puncture injury to his chest that caused a tension pneumothorax to form. This is a life-threatening condition because: A)Expired air exits the bleeding wound. B)Trapped, inspired air collapses the lung. C)The opposite lung hyperinflates. D)Blebs on the lung surface rupture.

B)Trapped, inspired air collapses the lung. Tension pneumothorax occurs when the intrapleuralpressure exceeds atmospheric pressure. It is a life-threatening condition and occurs when injury to the chest or respiratory structures permits air to enter but not leave the pleural space. Spontaneous pneumothorax occurs when an air-filled bleb, or blister, on the lung surface ruptures. Rupture of these blebs allows atmospheric air from the airways to enter the pleural cavity. This results in a rapid increase in pressure in the chest with a compression atelectasis of the unaffected lung.

In an ICU setting, one assessment that would lead the nurse to suspect shock has resulted in decrease blood flow to vital organs is: A)Warm legs with peripheral vasodilation B)Urine output less than 20 mL/hour C)Blood pressure staying in the 98/72 range for the past hour D)Sleepiness and difficulty to arouse without using painful stimuli

B)Urine output less than 20 mL/hour Continuous measurement of urine output is essential for assessing the circulatory status of a person in shock. Oliguria of 20 mL/hour or less indicates inadequate renal perfusion. Continuous measurement of urine output is essential for assessing the circulatory and volume status of the person in shock and monitoring the response to fluid replacement. As the shock progresses and blood flow to the brain decreases, restlessness is replaced by apathy and stupor. Sympathetic stimulation also leads to intense vasoconstriction of the skin vessels and activation of the sweat glands. As a result, the skin is cool and moist. There is an increase in heart rate, cool and clammy skin, a decrease in arterial blood pressure, and a decrease in urine output

A college student is lamenting the fact that she has developed a cold on the weekend prior to exam week. Which of the following statements shows that the student has an accurate understanding of her upper respiratory infection? A)"I'm just going to try to rest as much as I can until these bacteria clear up." B)"I think I'll go to the campus clinic and see if I can get a prescription for antibiotics." C)"I suppose I should have been washing my hands more in the past few days." D)"If I can just start some antihistamines as soon as possible I bet I'll get over this faster."

C)"I suppose I should have been washing my hands more in the past few days." Handwashing remains the most effective preventative measure against the common cold. Antihistamines have been shown to have no curative value, and antibiotics are ineffective because of the viral etiology of the common cold.

A 52-year-old man who is moderately obese has recently been diagnosed with hypertension by his primary care provider. Which of the client's following statements indicates a need for further health promotion teaching? A)"I've starting going to the gym before work three times a week." B)"I'm trying to cut back on the amount of salt that I cook with and add to my food." C)"I'm resolving to eat organic foods from now on and to drink a lot more water." D)"I'm planning to lose 15 pounds before the end of this year."

C)"I'm resolving to eat organic foods from now on and to drink a lot more water." Weight loss, exercise, and salt reduction are all useful strategies in the management of hypertension. An organic diet and increased fluid intake are not known to reduce blood pressure.

A client with persistent, primary hypertension remains apathetic about his high blood pressure, stating "I don't feel sick, and it doesn't seem to be causing me any problems that I can tell." How could the nurse best respondto this client's statement? A)"Actually, high blood pressure makes you very susceptible to getting diabetes in the future." B)"That's true, but it's an indicator that you're not taking very good care of yourself." C)"You may not sense any problems, but it really increases your risk of heart disease and stroke." D)"You're right, but it's still worthwhile to monitor it in case you do develop problems."

C)"You may not sense any problems, but it really increases your risk of heart disease and stroke." Hypertension is a highly significant risk factor for heart disease and stroke. It would be inappropriate to promote monitoring without promoting lifestyle modifications or other interventions to lower the client's blood pressure, or teaching the client about the deleterious effects of hypertension. It is likely unproductive to simply characterize the client's hypertension as demonstrating that he does not "take care" of himself. Hypertension is not a risk factor for the development of diabetes mellitus.

Which of the following assessment findings would be suggestive of cardiac tamponade? A)Increasing PaCO2and decreasing PaO2 B)Audible crackles on chest auscultation and presence of frothy sputum C)A 20 mm Hg drop in systolic blood pressure during respiration D)Normal ECG combined with complaints of chest pain and shortness of breath

C)A 20 mm Hg drop in systolic blood pressure during respiration A key diagnostic finding in cardiac tamponade is pulsus paradoxus, or an exaggeration of the normal variation in the systolic blood pressure, commonly defined as a 10 mm Hgor more fall in the systolic blood pressure, which occurs with inspiration. Worsening blood gases, chest secretions, and chest pain are not symptoms specific to cardiac tamponade. The client's ECG would not be normal.

A client with a diagnosis of heart failure has returned from a visit with his primary care provider with a prescription for a change in his daily medication regimen. Which of the following drugs is likely to improve the client's cardiac function by increasing the force and strength of ventricular contractions? A)A β-adrenergic blocker B)A diuretic C)A cardiac glycoside D)An ACE inhibitor

C)A cardiac glycoside Cardiac glycosides improve cardiac function by increasing the force and strength of ventricular contractions. β-Adrenergic blockers decrease left ventricular dysfunction associated with activation of the sympathetic nervous system. ACE inhibitors block the conversion of angiotensin I to II, whereas diuretics promote the excretion of fluid

Which of the following clients is at risk for developing a preventable disorder related to prolonged immobility? A)A middle-aged adult male diagnosed with bronchitis related to chronic smoking B)A young adult female diagnosed with sarcoidosis requiring corticosteroids to return her to remission C)A postsurgical client who is refusing to get out of bed and walk and will not wear those "uncomfortable elastic stocking" D)A sleep apnea client related to a history of smoking who utilizes a C-PAP machine every night at bedtime to maintain airway

C)A postsurgical client who is refusing to get out of bed and walk and will not wear those "uncomfortable elastic stocking" A lack of mobility can result in secondary atelectasis(through incomplete lung expansion) and pulmonary embolism (from deep vein thrombosis). This is not the case with the other listed disorders of ventilation and gas exchange.

Because of its location, the presence of an abdominal aortic aneurysm may first be manifested as: A)Constipation B)Indigestion C)A pulsating mass D)Midabdominal pain

C)A pulsating mass An abdominal aortic aneurysm may be noticed as a pulsating mass when the client is lying flat, once the aneurysm is rather large already. Although there may be pressure on the abdominal organs by the mass, it is not associated with constipation or indigestion. Severe midabdominal pain is a late sign of impending rupture.

A father experienced the onset of chest pain and dies suddenly. The family asks, "What caused him to die so suddenly?" The health care provider's reply that is most appropriate would be, "There's a high probability that your loved one developed an acute heart attack and experienced: A)Acute myocarditis." B)High troponin levels." C)Acute ventricular arrhythmia." D)Hypertrophic cardiomyopathy."

C)Acute ventricular arrhythmia." Sudden death from an acute myocardial infarctionin an adult is usually caused by fatal (ventricular) arrhythmias. Hypertrophic cardiomyopathy is the most common cause of sudden cardiac death in the young, since the disorder can be inherited as an autosomal dominant trait. Troponin is normally present in cardiac muscle; serum levels of troponin enzymes are diagnostic and will elevate within 3 hours of the acute event. Myocarditis is inflammation of the heart muscle and conduction system without evidence of myocardial infarction.

Which of the following clients should most likely be assessed for orthostatic hypotension? A)A 78-year-old woman who has begun complaining of frequent headaches unrelieved by over-the-counter analgesics B)A 65-year-old client whose vision has become much less acute in recent months and who has noticed swelling in her ankles C)An 80-year-old elderly client who has experienced two falls since admission while attempting to ambulate to the bathroom D)A 42-year-old client who has a history of poorly controlled type 1 diabetes

C)An 80-year-old elderly client who has experienced two falls since admission while attempting to ambulate to the bathroom Dizziness and syncope are characteristic signs and symptoms of orthostatichypotension, and both predispose an individual to falls; this is especially the case among older adults. Headaches, edema, diabetes, and vision changes are not associated with orthostatic hypotension.

A child's history of a recurrent sore throat followed by severe knee and ankle pain has resulted in a diagnostic workup and a diagnosis of rheumatic fever. What are the treatment priorities for this child?A)Cardiac catheterization and corticosteroid therapy B)Implanted pacemaker and β-adrenergic blockers C)Antibiotics and anti-inflammatory drugs D)Pain control and oxygen therapy

C)Antibiotics and anti-inflammatory drugs A diagnosis of rheumatic fever (RF) necessitates the use of antibiotics (usually penicillin) and anti-inflammatory drugs. These measures supersede the importance of pain control and oxygen therapy. Cardiac catheterization, corticosteroid therapy, pacemakers, and β-adrenergic blockers are not common treatment modalities for RF

A client has just been admitted to the postsurgical unit following a below-the-knee amputation. Which of the following measures should her care team prioritize to prevent atelectasis during the client's immediate recovery? A)Bedrest and supplementary oxygen by nasal cannula B)Administration of bronchodilators by nebulizer C)Deep-breathing exercises and early mobilization D)Adequate hydration and a high-humidity environment

C)Deep-breathing exercises and early mobilization Coughing and deep breathing and early ambulation decrease the likelihood of atelectasis developing in surgical clients; bedrest should be avoided when possible. Oxygen, bronchodilators, hydration, and high humidity do not prevent atelectasis.

Implantation of a pacemaker is most likely to benefit a client with which of the following cardiomyopathies? A)Myocarditis B)Takotsubo cardiomyopathy C)Dilated cardiomyopathy (DCM) D)Primary restrictive cardiomyopathy

C)Dilated cardiomyopathy (DCM) Arrhythmias and dysrhythmias are characteristic of DCM, often requiring the use of an implanted pacemaker or an implantable cardioverter-defibrillator. Implanted pacemakers are not noted to be among the common treatments for myocarditis, Takotsubo cardiomyopathy, or primary restrictive cardiomyopathy.

A client was in car accident client while not wearing a seatbelt and has sustained multiple rib fractures. During assessment, the nurse is having a hard time hearing heart sounds, and the client reports chest pain/pressure repeatedly. This client may be experiencing: A)Cardiomyopathy B)Pericarditis C)Pulmonary hypertension D)Pericardial effusion

D)Pericardial effusion Pericardial effusion is the accumulation of fluid in the pericardialcavity, usually as a result of an inflammatory reaction. It may develop with neoplasms, cardiac surgery, or trauma. Pericardial effusion exerts its effects through compression of the heart chambers. The normal pericardial space contains about 15 to 50 mL of fluid. Increases in the volume of this fluid, the rapidity with which it accumulates, and the elasticity of the pericardium determine the effect that the effusion has on cardiac function. This leads to cardiac standstill or failure. Pericardial sac thickening due to inflammation can restrict the heart, rather than to allow stretching. Rupture of the sac is pathologic, resulting in heart expansion. A friction rub sound (rubbing between the inflamed pericardial surfaces) is characteristic of acute pericarditis. Acquired cardiomyopathies include those that have their origin in the inflammatory process (e.g., myocarditis), pregnancy (peripartum cardiomyopathy), and stress (takotsubo cardiomyopathy). In congenital heart defects, in most cases, pulmonary vascular resistance is only slightly elevated during early infancy, and the major contribution to pulmonary hypertension is the increased blood flow.

A newly admitted critical head injury client presents to the neuro-ICU. The client is unresponsive to painful stimuli but able to breathe on his own. As the shift progresses, the nurses note a decrease in the client's respiratory effort. The client cannot maintain his O2 saturation above 70%. The nurses should anticipate assisting in beginning what type of pulmonary support? A)Increase oxygen level to 10 L/min. B)Begin Bi-PAP. C)Call respiratory therapy to suction the client. D)Prepare for mechanical ventilation.

D)Prepare for mechanical ventilation. When alveolar ventilation is inadequate to maintain PO2or PCO2levels because of respiratory or neurologic failure, mechanical ventilation may be lifesaving. Usually a nasotracheal, orotracheal, or tracheotomy tube is inserted into the trachea to provide the client with the airway needed for mechanical ventilation.

The pathogenic capacity of the tubercle bacillus is related to: A)Formation of a Ghon focus lesion B)Its inherent destructive capabilities C)Rapid viral replication in host cells D)The initiation of a cell-mediated immune response

D)The initiation of a cell-mediated immune response The pathogenesis of tuberculosis, in previously unexposed immunocompetentpeople, is a cell-mediated immune response that confers resistance to the organism and development of hypersensitivity to the tubercular antigens. Pathologic manifestations of tuberculosis, such as caseating granuloma and cavitation, are the result of the hypersensitivity reaction rather than its inherent destructive capabilities. In persons with intact cell-mediated immunity, the cell-mediated immune response results in the development of a granulomatous lesion, called a Ghon focus, that contains the tubercle bacilli, modified macrophages, and other immune cells.

With acute respiratory distress syndrome (ARDS), a client progressively increases his work of breathing. The physiological principle behind this respiratory distress is related to: A)Increases in left atrial pressure causing thickening of the lining of the pulmonary arteries B)The elevation of pulmonary venous pressure C)Structural abnormalities of pulmonary vessels with proliferation of the vessel intima D)The stiffening of the lung, making it more difficult to inflate

D)The stiffening of the lung, making it more difficult to inflate As the disease (ARDS) progresses, the work of breathing becomes greatly increased as the lung stiffens and becomes more difficult to inflate. There is increased intrapulmonary shunting of blood, impaired gas exchange, and hypoxemia despite high supplemental oxygen therapy. With pulmonary hypertension, there are continued increases in left atrial pressure, which can lead to medial hypertrophy and intimal thickening of the small pulmonary arteries, causing sustained hypertension. Pulmonary hypertension also causes an elevation of pulmonary venous pressure.

Endocarditis and rheumatic heart disease are both cardiac complications of systemic infections. Characteristics include a new or changed heart murmur caused by: A)Chronic atrial fibrillation B)Myocardial inflammation C)Left ventricle hypertrophy D)Vegetative valve destruction

D)Vegetative valve destruction Murmurs are sounds produced by blood flow through incompetent valves. Both infective endocarditis and carditis of rheumatic heart disease are characterized by growth of vegetation on valve leaflets, causing destruction, regurgitation, and murmur. Atrial fibrillation is a conduction disorder that impairs atrial emptying rather than valve function. Myocardial inflammation is present but does not cause murmurs. Valve dysfunctions can chronically decrease emptying and lead to left ventricular hypertrophy.

Severe shock can be followed by acute lung injury/acute respiratory distresssyndrome (ALI/ARDS) characterized by: A)Hyperventilation B)Excessive surfactant C)Hyperinflated alveolar sacs D)Ventilation-perfusion mismatch

D)Ventilation-perfusion mismatch Despite the delivery of high levels of oxygen using high-pressure mechanical ventilatory support and positive end-expiratory pressure, many persons with ALI/ARDS remain hypoxic, often with a fatal outcome. Arterial blood gas analysis establishes the presence of profound hypoxemia with hypercapnia, resulting from impaired matching of ventilation and perfusion and from the greatly reduced diffusion of blood gases across the thickened alveolar membranes. Abnormalities in the production, composition, and function of surfactant may contribute to alveolar collapse and gas exchange


Set pelajaran terkait

BA 3308 - Test 1 - Quiz Questions

View Set

MedSurg Exam 4 Review (Evolve Questions)

View Set

Fundamentals of Nursing Ch 20 PrepU 4

View Set

Chapter 9, (js), Systems of Equations and Inequalities

View Set